NUR2261 - Unit 3 - Labor Pain/Stages and Phases of Labor

Pataasin ang iyong marka sa homework at exams ngayon gamit ang Quizwiz!

The nurse would expect which maternal cardiovascular finding during labor? a. Increased cardiac output b. Decreased pulse rate c. Decreased white blood cell (WBC) count d. Decreased blood pressure

ANS: A During each contraction, 400 mL of blood is emptied from the uterus into the maternal vascular system. This increases cardiac output by about 51% above baseline pregnancy values at term. The heart rate increases slightly during labor. The WBC count can increase during labor. During the first stage of labor, uterine contractions cause systolic readings to increase by about 10 mm Hg. During the second stage, contractions may cause systolic pressures to increase by 30 mm Hg and diastolic readings to increase by 25 mm Hg.

38. What is an essential part of nursing care for the laboring woman? a. Helping the woman manage the pain b. Eliminating the pain associated with labor c. Sharing personal experiences regarding labor and delivery to decrease her anxiety d. Feeling comfortable with the predictable nature of intrapartum care

ANS: A Helping a woman manage the pain is an essential part of nursing care because pain is an expected part of normal labor and cannot be fully relieved. Decreasing anxiety is important however, managing pain is a top priority. The labor nurse should consistently deliver care based on the standard of care related to the maternity patient. Because of the unpredictable nature of labor, the nurse should always be alert for unanticipated events.

9. Which presentation is described accurately in terms of both presenting part and frequency of occurrence? a. Cephalic: occiput at least 95% b. Breech: sacrum 10% to 15% c. Shoulder: scapula 10% to 15% d. Cephalic: cranial 80% to 85%

ANS: A In cephalic presentations (head first), the presenting part is the occiput. this occurs in 96% of births. In a breech birth, the sacrum emerges first, this occurs in about 3% of births. In shoulder presentations, the scapula emerges first, this occurs in only 1% of births.

7. The factors that affect the process of labor and birth, known commonly as thefive Ps, include all except: a. Passenger. c. Powers. b. Passageway. d. Pressure.

ANS: D The five Ps are passenger (fetus and placenta), passageway (birth canal), powers (contractions), position of the mother, and psychologic response.

28. If a woman complains of back labor pain, the nurse could best suggest that she: a. Lie on her back for a while with her knees bent. b. Do less walking around. c. Take some deep, cleansing breaths. d. Lean over a birth ball with her knees on the floor.

ANS: D The hands-and-knees position, with or without the aid of a birth ball, should help with the back pain. The supine position should be discouraged. Walking generally is encouraged.

A client who is 25 weeks' pregnant and who is being treated with atenolol reports labor pain. Which medication would the nurse anticipate incorporating into the plan of care? 1 Sucralfate 2 Nifedipine 3 Indomethacin 4 Dexamethasone

2 Nifedipine inhibits myometrial activity by blocking calcium reflux. This action helps reduce preterm labor. Indomethacin is commonly used to treat preterm labor. However, concomitant use of atenolol and indomethacin may increase maternal and fetal risk. Dexamethasone is administered if neither indomethacin nor nifedipine is effective. Sucralfate is used to protect the stomach from gastrointestinal issues associated with indomethacin.

24. In documenting labor experiences, nurses should know that a uterine contraction is described according to all these characteristics except: a. Frequency (how often contractions occur). b. Intensity (the strength of the contraction at its peak). c. Resting tone (the tension in the uterine muscle). d. Appearance (shape and height).

ANS: D Uterine contractions are described in terms of frequency, intensity, duration, and resting tone.

23. Leopold maneuvers would be an inappropriate method of assessment to determine: a. Gender of the fetus. b. Number of fetuses. c. Fetal lie and attitude. d. Degree of the presenting parts descent into the pelvis.

ANS: A Leopold maneuvers help identify the number of fetuses, the fetal lie and attitude, and the degree of descent of the presenting part into the pelvis. The gender of the fetus is not a goal of the examination at this time.

17. After an emergency birth, the nurse encourages the woman to breastfeed her newborn. The primary purpose of this activity is to: a. Facilitate maternal-newborn interaction. b. Stimulate the uterus to contract. c. Prevent neonatal hypoglycemia. d. Initiate the lactation cycle.

ANS: B Stimulation of the nipples through breastfeeding or manual stimulation causes the release of oxytocin and prevents maternal hemorrhage. Breastfeeding facilitates maternal-newborn interaction, but it is not the primary reason a woman is encouraged to breastfeed after an emergency birth. The primary intervention for preventing neonatal hypoglycemia is thermoregulation. Cold stress can result in hypoglycemia. The woman is encouraged to breastfeed after an emergency birth to stimulate the release of oxytocin, which prevents hemorrhage. Breastfeeding is encouraged to initiate the lactation cycle, but it is not the primary reason for this activity after an emergency birth.

31. The laboring woman who imagines her body opening to let the baby out is using a mental technique called: a. Dissociation. c. Imagery. b. Effleurage. d. Distraction.

ANS: C Imagery is a technique of visualizing images that will assist the woman in coping with labor. Dissociation helps the woman learn to relax all muscles except those that are working. Effleurage is self-massage. Distraction can be used in the early latent phase by having the woman engage in another activity.

26. The nurse caring for the woman in labor should understand that increased variability of the fetal heart rate may be caused by: a. Narcotics. c. Methamphetamines. b. Barbiturates. d. Tranquilizers.

ANS: C Narcotics, barbiturates, and tranquilizers may be causes of decreased variability methamphetamines may cause increased variability.

19. A normal uterine activity pattern in labor is characterized by: a. Contractions every 2 to 5 minutes. b. Contractions lasting about 2 minutes. c. Contractions about 1 minute apart. d. A contraction intensity of about 1000 mm Hg with relaxation at 50 mm Hg.

ANS: A Contractions normally occur every 2 to 5 minutes and last less than 90 seconds (intensity 800 mm Hg) with about 30 seconds in between (20 mm Hg or less).

30. Which method of pain management is safest for a gravida 3 para 2 admitted at 8 cm cervical dilation? a. Epidural anesthesia c. Spinal block b. Narcotics d. Breathing and relaxation techniques

ANS: D Nonpharmacologic methods of pain management may be the best option for a woman in advanced labor. It is unlikely that enough time remains to administer epidural or spinal anesthesia. A narcotic given at this time may reach its peak about the time of birth and result in respiratory depression in the newborn.

8. A woman is experiencing back labor and complains of intense pain in her lower back. An effective relief measure would be to use: a. Counterpressure against the sacrum. b. Pant-blow (breaths and puffs) breathing techniques. c. Effleurage. d. Conscious relaxation or guided imagery.

ANS: A Counterpressure is steady pressure applied by a support person to the sacral area with the fist or heel of the hand. This technique helps the woman cope with the sensations of internal pressure and pain in the lower back. The pain management techniques of pant-blow, effleurage, and conscious relaxation or guided imagery are usually helpful for contractions per the gate-control theory.

43. At 1 minute after birth, the nurse assesses the newborn to assign an Apgar score. The apical heart rate is 110 bpm, and the infant is crying vigorously with the limbs flexed. The infants trunk is pink, but the hands and feet are blue. What is the correct Apgar score for this infant? a. 7 c. 9 b. 8 d. 10

ANS: C The Apgar score is 9 because 1 point is deducted from the total score of 10 for the infants blue hands and feet. The baby received 2 points for each of the categories except color. Because the infants hands and feet were blue, this category is given a grade of 1.

A client with hypertension has labor pains before the 35th week of gestation. Which pharmacologic intervention does the nurse anticipate to be beneficial for this client? 1 Preparing the client for an abortion 2 Administer terbutaline 3 Administer magnesium sulfate 4 Administer sedatives and maintain hydration

Answer: 3 Magnesium sulfate is the drug of choice for maintaining pregnancy in preterm labor in hypertensive clients. An abortion is advisable if the preterm labor starts before the 20th week of gestation because the fetus may be nonviable. Terbutaline is used to maintain pregnancy in preterm labor by relaxing the uterine smooth muscle. The use of sedatives and body hydration measure are nonpharmacologic interventions to maintain pregnancy in preterm labor.

A pregnant client has labor pains; however, upon assessment, the nurse finds that the cervix is not dilated. Which medication would the nurse identify that could be used to promote labor? 1 Oxytocin 2 Nifedipine 3 Dinoprostone 4 Methylergonovine

3 DinoprostoneDinoprostone induces cervical ripening. This action helps in the induction of labor at term. Oxytocin enhances labor when uterine contractions are weak and ineffective. Nifedipine is a calcium channel blocker used to maintain pregnancy during preterm labor. Methylergonovine reduces postpartum hemorrhage.

35. As the United States and Canada continue to become more culturally diverse, it is increasingly important for the nursing staff to recognize a wide range of varying cultural beliefs and practices. Nurses need to develop respect for these culturally diverse practices and learn to incorporate these into a mutually agreed on plan of care. Although it is common practice in the United States for the father of the baby to be present at the birth, in many societies this is not the case. When implementing care, the nurse would anticipate that a woman from which country would have the father of the baby in attendance? a. Mexico c. Iran b. China d. India

ANS: A A woman from Mexico may be stoic about discomfort until the second stage, at which time she will request pain relief. Fathers and female relatives are usually in attendance during the second stage of labor. The father of the baby is expected to provide encouragement, support, and reassurance that all will be well. Fathers are usually not present in China. The Iranian father will not be present. Female support persons and female care providers are preferred. For many, a male caregiver is unacceptable. The father is usually not present in India, but female relatives are usually present. Natural childbirth methods are preferred.

Which description of the four stages of labor is correct for both definition and duration? a. First stage: onset of regular uterine contractions to full dilation less than 1 hour to 20 hours

ANS: A Full dilation may occur in less than 1 hour, but in first-time pregnancies it can take up to 20 hours. The second stage extends from full dilation to birth and takes an average of 20 to 50 minutes, although 2 hours is still considered normal. The third stage extends from birth to expulsion of the placenta and usually takes a few minutes. The fourth stage begins after expulsion of the placenta and lasts until homeostasis is reestablished (about 2 hours).

31. A means of controlling the birth of the fetal head with a vertex presentation is: a. The Ritgen maneuver. c. The lithotomy position. b. Fundal pressure. d. The De Lee apparatus.

ANS: A The Ritgen maneuver extends the head during the actual birth and protects the perineum. Gentle, steady pressure against the fundus of the uterus facilitates vaginal birth. The lithotomy position has been commonly used in Western cultures, partly because it is convenient for the health care provider. The De Lee apparatus is used to suction fluid from the infants mouth.

To care for a laboring woman adequately, the nurse understands that the __________ stage of labor varies the most in length? a. First c. Third b. Second d. Fourth

ANS: A The first stage of labor is considered to last from the onset of regular uterine contractions to full dilation of the cervix. The first stage is much longer than the second and third stages combined. In a first-time pregnancy the first stage of labor can take up to 20 hours. The second stage of labor lasts from the time the cervix is fully dilated to the birth of the fetus. The average length is 20 minutes for a multiparous woman and 50 minutes for a nulliparous woman. The third stage of labor lasts from the birth of the fetus until the placenta is delivered. This stage may be as short as 3 minutes or as long as 1 hour. The fourth stage of labor, recovery, lasts about 2 hours after delivery of the placenta.

41. A laboring woman is lying in the supine position. The most appropriate nursing action at this time is to: a. Ask her to turn to one side. b. Elevate her feet and legs. c. Take her blood pressure. d. Determine whether fetal tachycardia is present.

ANS: A The womans supine position may cause the heavy uterus to compress her inferior vena cava, thus reducing blood return to her heart and reducing placental blood flow. Elevating her legs will not relieve the pressure from the inferior vena cava. If the woman is allowed to stay in the supine position and blood flow to the placental is reduced significantly, fetal tachycardia may occur. The most appropriate nursing action is to prevent this from occurring by turning the woman to her side. Blood pressure readings may be obtained when the patient is in the appropriate and safest position.

27. With regard to a womans intake and output during labor, nurses should be aware that: a. The tradition of restricting the laboring woman to clear liquids and ice chips is being challenged because regional anesthesia is used more often than general anesthesia. b. Intravenous (IV) fluids usually are necessary to ensure that the laboring woman stays hydrated. c. Routine use of an enema empties the rectum and is very helpful for producing a clean, clear delivery. d. When a nulliparous woman experiences the urge to defecate, it often means birth will follow quickly.

ANS: A Women are awake with regional anesthesia and are able to protect their own airway, which reduces the worry over aspiration. Routine IV fluids during labor are unlikely to be beneficial and may be harmful. Routine use of an enema is at best ineffective and may be harmful. A multiparous woman may feel the urge to defecate and it may mean birth will follow quickly, but not for a first-timer.

MULTIPLE RESPONSE 45. Women who have participated in childbirth education classes often bring a birth bag or Lamaze bag with them to the hospital. These items often assist in reducing stress and providing comfort measures. The nurse caring for women in labor should be aware of common items that a client may bring, including (Select all that apply): a. Rolling pin. b. Tennis balls c. Pillow. d. Stuffed animal or photo e. Candles.

ANS: A, B, C, D The rolling pin and tennis balls are used to provide counterpressure, especially if the woman is experiencing back labor. Although the facility has plenty of pillows, when the client brings her own, it is a reminder of home and provides added comfort. A stuffed animal or framed photo can be used to provide a focal point during contractions. Although many women find the presence of candles conducive to creating calm and relaxing surroundings, these are not suitable for a hospital birthing room environment. Oxygen may be in use, resulting in a fire hazard. Flameless candles are often sold in hospital gift shops. It is also important for the nurse to orient the patient and her family to the call bell and light switches to familiarize herself with the environment.

33. Concerning the third stage of labor, nurses should be aware that: a. The placenta eventually detaches itself from a flaccid uterus. b. An expectant or active approach to managing this stage of labor reduces the risk of complications. c. It is important that the dark, roughened maternal surface of the placenta appear before the shiny fetal surface. d. The major risk for women during the third stage is a rapid heart rate.

ANS: B Active management facilitates placental separation and expulsion, thus reducing the risk of complications. The placenta cannot detach itself from a flaccid (relaxed) uterus. Which surface of the placenta comes out first is not clinically important. The major risk for women during the third stage of labor is postpartum hemorrhage.

39. A woman who is gravida 3 para 2 enters the intrapartum unit. The most important nursing assessments are: a. Contraction pattern, amount of discomfort, and pregnancy history. b. Fetal heart rate, maternal vital signs, and the womans nearness to birth. c. Identification of ruptured membranes, the womans gravida and para, and her support person. d. Last food intake, when labor began, and cultural practices the couple desires.

ANS: B All options describe relevant intrapartum nursing assessments however, this focused assessment has priority. If the maternal and fetal conditions are normal and birth is not imminent, other assessments can be performed in an unhurried manner. This includes: gravida, para, support person, pregnancy history, pain assessment, last food intake, and cultural practices.

26. Under which circumstance would it be unnecessary for the nurse to perform a vaginal examination? a. An admission to the hospital at the start of labor b. When accelerations of the fetal heart rate (FHR) are noted c. On maternal perception of perineal pressure or the urge to bear down d. When membranes rupture

ANS: B An accelerated FHR is a positive sign however, variable decelerations merit a vaginal examination. Vaginal examinations should be performed when the woman is admitted, when she perceives perineal pressure or the urge to bear down, when her membranes rupture, when a significant change in her uterine activity has occurred, or when variable decelerations of the FHR are noted.

The slight overlapping of cranial bones or shaping of the fetal head during labor is called: a. Lightening. c. Ferguson reflex. b. Molding. d. Valsalva maneuver.

ANS: B Fetal head formation is called molding. Molding also permits adaptation to various diameters of the maternal pelvis. Lightening is the mothers sensation of decreased abdominal distention, which usually occurs the week before labor. The Ferguson reflex is the contraction urge of the uterus after stimulation of the cervix. The Valsalva maneuver describes conscious pushing during the second stage of labor.

The nurse has received report regarding her patient in labor. The womans last vaginal examination was recorded as 3 cm, 30%, and -2-2. The nurses interpretation of this assessment is that: a. The cervix is effaced 3 cm, it is dilated 30%, and the presenting part is 2 cm above the ischial spines. b. The cervix is 3 cm dilated, it is effaced 30%, and the presenting part is 2 cm above the ischial spines. c. The cervix is effaced 3 cm, it is dilated 30%, and the presenting part is 2 cm below the ischial spines. d. The cervix is dilated 3 cm, it is effaced 30%, and the presenting part is 2 cm below the ischial spines.

ANS: B The correct description of the vaginal examination for this woman in labor is the cervix is 3 cm dilated, it is effaced 30%, and the presenting part is 2 cm above the ischial spines. The sterile vaginal examination is recorded as centimeters of cervical dilation, percentage of cervical dilation, and the relationship of the presenting part to the ischial spines (either above or below).

36. A patient whose cervix is dilated to 5 cm is considered to be in which phase of labor? a. Latent phase c. Second stage b. Active phase d. Third stage

ANS: B The latent phase is from the beginning of true labor until 3 cm of cervical dilation. The active phase of labor is characterized by cervical dilation of 4 to 7 cm. The second stage of labor begins when the cervix is completely dilated until the birth of the baby. The third stage of labor is from the birth of the baby until the expulsion of the placenta. This patient is in the active phase of labor.

34. For women who have a history of sexual abuse, a number of traumatic memories may be triggered during labor. The woman may fight the labor process and react with pain or anger. Alternately, she may become a passive player and emotionally absent herself from the process. The nurse is in a unique position of being able to assist the client to associate the sensations of labor with the process of childbirth and not the past abuse. The nurse can implement a number of care measures to help the client view the childbirth experience in a positive manner. Which intervention would be key for the nurse to use while providing care? a. Telling the client to relax and that it wont hurt much b. Limiting the number of procedures that invade her body c. Reassuring the client that as the nurse you know what is best d. Allowing unlimited care providers to be with the client

ANS: B The number of invasive procedures such as vaginal examinations, internal monitoring, and intravenous therapy should be limited as much as possible. The nurse should always avoid words and phrases that may result in the clients recalling the phrases of her abuser (e.g., Relax, this wont hurt or Just open your legs.) The womans sense of control should be maintained at all times. The nurse should explain procedures at the clients pace and wait for permission to proceed. Protecting the clients environment by providing privacy and limiting the number of staff who observe the client will help to make her feel safe.

30. Nurses alert to signs of the onset of the second stage of labor can be certain that this stage has begun when: a. The woman has a sudden episode of vomiting. b. The nurse is unable to feel the cervix during a vaginal examination. c. Bloody show increases. d. The woman involuntarily bears down.

ANS: B The only certain objective sign that the second stage has begun is the inability to feel the cervix because it is fully dilated and effaced. Vomiting, an increase in bloody show, and involuntary bearing down are only suggestions of second-stage labor.

11. As relates to fetal positioning during labor, nurses should be aware that: a. Position is a measure of the degree of descent of the presenting part of the fetus through the birth canal. b. Birth is imminent when the presenting part is at +4 to +5 cm below the spine. c. The largest transverse diameter of the presenting part is the suboccipitobregmatic diameter. d. Engagement is the term used to describe the beginning of labor.

ANS: B The station of the presenting part should be noted at the beginning of labor so that the rate of descent can be determined. Position is the relation of the presenting part of the fetus to the four quadrants of the mothers pelvis station is the measure of degree of descent. The largest diameter usually is the biparietal diameter. The suboccipitobregmatic diameter is the smallest, although one of the most critical. Engagement often occurs in the weeks just before labor in nulliparas and before or during labor in multiparas.

42. Which nursing assessment indicates that a woman who is in second-stage labor is almost ready to give birth? a. The fetal head is felt at 0 station during vaginal examination. b. Bloody mucus discharge increases. c. The vulva bulges and encircles the fetal head. d. The membranes rupture during a contraction.

ANS: C A bulging vulva that encircles the fetal head describes crowning, which occurs shortly before birth. Birth of the head occurs when the station is +4. A 0 station indicates engagement. Bloody show occurs throughout the labor process and is not an indication of an imminent birth. Rupture of membranes can occur at any time during the labor process and does not indicate an imminent birth.

2. When assessing a woman in labor, the nurse is aware that the relationship of the fetal body parts to one another is called fetal: a. Lie. c. Attitude. b. Presentation. d. Position.

ANS: C Attitude is the relation of the fetal body parts to one another. Lie is the relation of the long axis (spine) of the fetus to the long axis (spine) of the mother. Presentation refers to the part of the fetus that enters the pelvic inlet first and leads through the birth canal during labor at term. Position is the relation of the presenting part to the four quadrants of the mothers pelvis.

While providing care to a patient in active labor, the nurse should instruct the woman that: a. The supine position commonly used in the United States increases blood flow. b. The all fours position, on her hands and knees, is hard on her back. c. Frequent changes in position will help relieve her fatigue and increase her comfort. d. In a sitting or squatting position, her abdominal muscles will have to work harder.

ANS: C Frequent position changes relieve fatigue, increase comfort, and improve circulation. Blood flow can be compromised in the supine position any upright position benefits cardiac output. The all fours position is used to relieve backache in certain situations. In a sitting or squatting position, the abdominal muscles work in greater harmony with uterine contractions.

44. The nurse thoroughly dries the infant immediately after birth primarily to: a. Stimulate crying and lung expansion. b. Remove maternal blood from the skin surface. c. Reduce heat loss from evaporation. d. Increase blood supply to the hands and feet.

ANS: C Infants are wet with amniotic fluid and blood at birth, and this accelerates evaporative heat loss. The primary purpose of drying the infant is to prevent heat loss. Rubbing the infant does stimulate crying however, it is not the main reason for drying the infant. This process does not remove all the maternal blood.

25. Because the risk for childbirth complications may be revealed, nurses should know that the point of maximal intensity (PMI) of the fetal heart tone (FHT) is: a. Usually directly over the fetal abdomen. b. In a vertex position heard above the mothers umbilicus. c. Heard lower and closer to the midline of the mothers abdomen as the fetus descends and rotates internally. d. In a breech position heard below the mothers umbilicus.

ANS: C Nurses should be prepared for the shift. The PMI of the FHT usually is directly over the fetal back. In a vertex position it is heard below the mothers umbilicus. In a breech position it is heard above the mothers umbilicus.

29. Which description of the phases of the second stage of labor is accurate? a. Latent phase: Feeling sleepy, fetal station 2+ to 4+, duration 30 to 45 minutes b. Active phase: Overwhelmingly strong contractions, Ferguson reflux activated, duration 5 to 15 minutes c. Descent phase: Significant increase in contractions, Ferguson reflux activated, average duration varied d. Transitional phase: Woman laboring down, fetal station 0, duration 15 minutes

ANS: C The descent phase begins with a significant increase in contractions the Ferguson reflex is activated, and the duration varies, depending on a number of factors. The latent phase is the lull, or laboring down, period at the beginning of the second stage. It lasts 10 to 30 minutes on average. The second stage of labor has no active phase. The transition phase is the final phase in the second stage of labor contractions are strong and painful.

When assessing the fetus using Leopold maneuvers, the nurse feels a round, firm, movable fetal part in the fundal portion of the uterus and a long, smooth surface in the mothers right side close to midline. What is the likely position of the fetus? a. ROA c. RSA b. LSP d. LOA

ANS: C The fetus is positioned anteriorly in the right side of the maternal pelvis with the sacrum as the presenting part. RSA is the correct three-letter abbreviation to indicate this fetal position. The first letter indicates the presenting part in either the right or left side of the maternal pelvis. The second letter indicates the anatomic presenting part of the fetus. The third letter stands for the location of the presenting part in relation to the anterior, posterior, or transverse portion of the maternal pelvis. Palpation of a round, firm fetal part in the fundal portion of the uterus would be the fetal head, indicating that the fetus is in a breech position with the sacrum as the presenting part in the maternal pelvis. Palpation of the fetal spine along the mothers right side denotes the location of the presenting part in the mothers pelvis. The ability to palpate the fetal spine indicates that the fetus is anteriorly positioned in the maternal pelvis.

10. With regard to factors that affect how the fetus moves through the birth canal, nurses should be aware that: a. The fetal attitude describes the angle at which the fetus exits the uterus. b. Of the two primary fetal lies, the horizontal lie is that in which the long axis of the fetus is parallel to the long axis of the mother. c. The normal attitude of the fetus is called general flexion. d. The transverse lie is preferred for vaginal birth.

ANS: C The normal attitude of the fetus is general flexion. The fetal attitude is the relation of fetal body parts to one another. The horizontal lie is perpendicular to the mother in the longitudinal (or vertical) lie the long axes of the fetus and the mother are parallel. Vaginal birth cannot occur if the fetus stays in a transverse lie.

32. Which collection of risk factors most likely would result in damaging lacerations (including episiotomies)? a. A dark-skinned woman who has had more than one pregnancy, who is going through prolonged second-stage labor, and who is attended by a midwife b. A reddish-haired mother of two who is going through a breech birth c. A dark-skinned, first-time mother who is going through a long labor d. A first-time mother with reddish hair whose rapid labor was overseen by an obstetrician

ANS: D Reddish-haired women have tissue that is less distensible than that of darker-skinned women and therefore may have less efficient healing. First time mothers are also more at risk, especially with breech births, long second-stage labors, or rapid labors in which there is insufficient time for the perineum to stretch. The rate of episiotomies is higher when obstetricians rather than midwives attend births.

40. A primigravida at 39 weeks of gestation is observed for 2 hours in the intrapartum unit. The fetal heart rate has been normal. Contractions are 5 to 9 minutes apart, 20 to 30 seconds in duration, and of mild intensity. Cervical dilation is 1 to 2 cm and uneffaced (unchanged from admission). Membranes are intact. The nurse should expect the woman to be: a. Admitted and prepared for a cesarean birth. b. Admitted for extended observation. c. Discharged home with a sedative. d. Discharged home to await the onset of true labor.

ANS: D This situation describes a woman with normal assessments who is probably in false labor and will likely not deliver rapidly once true labor begins. There is no indication that further assessments or observations are indicated therefore, the patient will be discharged along with instructions to return when contractions increase in intensity and frequency. Neither a cesarean birth nor a sedative is required at this time.

37. The primary difference between the labor of a nullipara and that of a multipara is the: a. Amount of cervical dilation. c. Level of pain experienced. b. Total duration of labor. d. Sequence of labor mechanisms. ANS: B

Multiparas usually labor more quickly than nulliparas, thus making the total duration of their labor shorter. Cervical dilation is the same for all labors. The level of pain is individual to the woman, not to the number of labors she has experienced. The sequence of labor mechanisms remains the same with all labors.

18. A pregnant woman is in her third trimester. She asks the nurse to explain how she can tell true labor from false labor. The nurse would explain that true labor contractions: a. Increase with activity such as ambulation. b. Decrease with activity. c. Are always accompanied by the rupture of the bag of waters. d. Alternate between a regular and an irregular pattern.

ANS: A True labor contractions become more intense with walking. False labor contractions often stop with walking or position changes. Rupture of membranes may occur before or during labor. True labor contractions are regular.

19. With regard to breathing techniques during labor, maternity nurses should understand that: a. Breathing techniques in the first stage of labor are designed to increase the size of the abdominal b. By the time labor has begun, it is too late for instruction in breathing and relaxation. c. Controlled breathing techniques are most difficult near the end of the second stage of labor. d. The patterned-paced breathing technique can help prevent hyperventilation.

ANS: A First-stage techniques promote relaxation of abdominal muscles, thereby increasing the size of the abdominal cavity. Instruction in simple breathing and relaxation techniques early in labor is possible and effective. Controlled breathing techniques are most difficult in the transition phase at the end of the first stage of labor when the cervix is dilated 8 to 10 cm. Patterned-paced breathing sometimes can lead to hyperventilation.

29. The nerve block used in labor that provides anesthesia to the lower vagina and perineum is called: a. An epidural. c. A local. b. A pudendal. d. A spinal block.

ANS: B A pudendal block anesthetizes the lower vagina and perineum to provide anesthesia for an episiotomy and use of low forceps if needed. An epidural provides anesthesia for the uterus, perineum, and legs. A local provides anesthesia for the perineum at the site of the episiotomy. A spinal block provides anesthesia for the uterus, perineum, and down the legs.

2. Nursing care measures are commonly offered to women in labor. Which nursing measure reflects application of the gate-control theory? a. Massaging the womans back b. Changing the womans position c. Giving the prescribed medication d. Encouraging the woman to rest between contractions

ANS: A According to the gate-control theory, pain sensations travel along sensory nerve pathways to the brain, but only a limited number of sensations, or messages, can travel through these nerve pathways at one time. Distraction techniques such as massage or stroking, music, focal points, and imagery reduce or completely block the capacity of nerve pathways to transmit pain. These distractions are thought to work by closing down a hypothetic gate in the spinal cord and thus preventing pain signals from reaching the brain. The perception of pain is thereby diminished. Changing the womans position, giving prescribed medication, and encouraging rest do not reduce or block the capacity of nerve pathways to transmit pain using the gate-control theory. Test Bank - Maternal Child Nursing Care by Perry (6th Edition, 2017) 217

7. The nurse providing care for the laboring woman should understand that amnioinfusion is used to treat: a. Variable decelerations. c. Fetal bradycardia. b. Late decelerations. d. Fetal tachycardia.

ANS: A Amnioinfusion is used during labor either to dilute meconium-stained amniotic fluid or to supplement the amount of amniotic fluid to reduce the severity of variable decelerations caused by cord compression. Amnioinfusion has no bearing on late decelerations, fetal bradycardia, or fetal tachycardia alterations in fetal heart rate (FHR) tracings.

28. The nurse providing newborn stabilization must be aware that the primary side effect of maternal narcotic analgesia in the newborn is: a. Respiratory depression. c. Acrocyanosis. b. Bradycardia. d. Tachypnea.

ANS: A An infant delivered within 1 to 4 hours of maternal analgesic administration is at risk for respiratory depression from the sedative effects of the narcotic. Bradycardia is not the anticipated side effect of maternal analgesics. Acrocyanosis is an expected finding in a newborn and is not related to maternal analgesics. The infant who is having a side effect to maternal analgesics normally would have a decrease in respirations, not an increase.

22. With regard to nerve block analgesia and anesthesia, nurses should be aware that: a. Most local agents are related chemically to cocaine and end in the suffix -caine. b. Local perineal infiltration anesthesia is effective when epinephrine is added, but it can be injected only once. c. A pudendal nerve block is designed to relieve the pain from uterine contractions. d. A pudendal nerve block, if done correctly, does not significantly lessen the bearing-down reflex.

ANS: A Common agents include lidocaine and chloroprocaine. Injections can be repeated to prolong the anesthesia. A pudendal nerve block relieves pain in the vagina, vulva, and perineum but not the pain from uterine contractions, and it lessens or shuts down the bearing-down reflex.

27. Which deceleration of the fetal heart rate would not require the nurse to change the maternal position? a. Early decelerations b. Late decelerations c. Variable decelerations d. It is always a good idea to change the womans position.

ANS: A Early decelerations (and accelerations) generally do not need any nursing intervention. Late decelerations suggest that the nurse should change the maternal position (lateral) variable decelerations also require a maternal position change (side to side). Although changing positions throughout labor is recommended, it is not required in response to early decelerations.

3. The nurse caring for the laboring woman should understand that early decelerations are caused by: a. Altered fetal cerebral blood flow. c. Uteroplacental insufficiency. b. Umbilical cord compression. d. Spontaneous rupture of membranes.

ANS: A Early decelerations are the fetuss response to fetal head compression. Variable decelerations are associated with umbilical cord compression. Late decelerations are associated with uteroplacental insufficiency. Spontaneous rupture of membranes has no bearing on the fetal heart rate unless the umbilical cord prolapses, which would result in variable or prolonged bradycardia.

4. The nurse providing care for the laboring woman comprehends that accelerations with fetal movement: a. Are reassuring. b. Are caused by umbilical cord compression. c. Warrant close observation. d. Are caused by uteroplacental insufficiency.

ANS: A Episodic accelerations in the fetal heart rate (FHR) occur during fetal movement and are indications of fetal well-being. Umbilical cord compression results in variable decelerations in the FHR. Accelerations in the FHR are an indication of fetal well-being and do not warrant close observation. Uteroplacental insufficiency would result in late decelerations in the FHR.

13. Fetal well-being during labor is assessed by: a. The response of the fetal heart rate (FHR) to uterine contractions (UCs). b. Maternal pain control. c. Accelerations in the FHR. d. An FHR above 110 beats/min.

ANS: A Fetal well-being during labor can be measured by the response of the FHR to UCs. In general, reassuring FHR patterns are characterized by an FHR baseline in the range of 110 to 160 beats/min with no periodic changes, a moderate baseline variability, and accelerations with fetal movement. Maternal pain control is not the measure used to determine fetal well-being in labor. Although FHR accelerations are a reassuring pattern, they are only one component of the criteria by which fetal well-being is assessed. Although an FHR above 110 beats/min may be reassuring, it is only one component of the criteria by which fetal well-being is assessed. More information would be needed to determine fetal well-being.

33. Which maternal condition is considered a contraindication for the application of internal monitoring devices? a. Unruptured membranes c. External monitors in current use b. Cervix dilated to 4 cm d. Fetus with a known heart defect

ANS: A In order to apply internal monitoring devices, the membranes must be ruptured. Cervical dilation of 4 cm permits the insertion of fetal scalp electrodes and intrauterine catheter. The external monitor can be discontinued after the internal ones are applied. A compromised fetus should be monitored with the most accurate monitoring devices.

2. While evaluating an external monitor tracing of a woman in active labor, the nurse notes that the fetal heart rate (FHR) for five sequential contractions begins to decelerate late in the contraction, with the nadir of the decelerations occurring after the peak of the contraction. The nurses first priority is to: a. Change the womans position. c. Assist with amnioinfusion. b. Notify the care provider. d. Insert a scalp electrode.

ANS: A Late decelerations may be caused by maternal supine hypotension syndrome. They usually are corrected when the woman turns on her side to displace the weight of the gravid uterus from the vena cava. If the fetus does not respond to primary nursing interventions for late decelerations, the nurse would continue with subsequent intrauterine resuscitation measures, including notifying the care provider. An amnioinfusion may be used to relieve pressure on an umbilical cord that has not prolapsed. The FHR pattern associated with this situation most likely reveals variable deceleration. A fetal scalp electrode would provide accurate data for evaluating the well-being of the fetus however, this is not a nursing intervention that would alleviate late decelerations, nor is it the nurses first priority.

9. The nurse caring for a laboring woman is aware that maternal cardiac output can be increased by: a. Change in position. c. Regional anesthesia. b. Oxytocin administration. d. Intravenous analgesic.

ANS: A Maternal supine hypotension syndrome is caused by the weight and pressure of the gravid uterus on the ascending vena cava when the woman is in a supine position. This reduces venous return to the womans heart, as well as cardiac output, and subsequently reduces her blood pressure. The nurse can encourage the woman to change positions and avoid the supine position. Oxytocin administration, regional anesthesia, and intravenous analgesic may reduce maternal cardiac output.

3. A woman in active labor receives an analgesic opioid agonist. Which medication relieves severe, persistent, or recurrent pain, creates a sense of well-being, overcomes inhibitory factors, and may even relax the cervix but should be used cautiously in women with cardiac disease? a. Meperidine (Demerol) c. Butorphanol tartrate (Stadol) b. Promethazine (Phenergan) d. Nalbuphine (Nubain)

ANS: A Meperidine is the most commonly used opioid agonist analgesic for women in labor throughout the world. It overcomes inhibitory factors in labor and may even relax the cervix. Because tachycardia is a possible adverse reaction, meperidine is used cautiously in women with cardiac disease. Phenergan is an ataractic (tranquilizer) that may be used to augment the desirable effects of the opioid analgesics but has few of the undesirable effects of those drugs. Stadol and Nubain are opioid agonist-antagonist analgesics.

16. Perinatal nurses are legally responsible for: a. Correctly interpreting fetal heart rate (FHR) patterns, initiating appropriate nursing interventions, and documenting the outcomes. b. Greeting the client on arrival, assessing her, and starting an intravenous line. c. Applying the external fetal monitor and notifying the care provider. d. Making sure that the woman is comfortable.

ANS: A Nurses who care for women during childbirth are legally responsible for correctly interpreting FHR patterns, initiating appropriate nursing interventions based on those patterns, and documenting the outcomes of those interventions. Greeting the client, assessing her, and starting an IV applying the external fetal monitor and notifying the care provider and making sure the woman is comfortable may be activities that a nurse performs, but they are not activities for which the nurse is legally responsible.

15. When assessing a multiparous woman who has just given birth to an 8-pound boy, the nurse notes that the womans fundus is firm and has become globular in shape. A gush of dark red blood comes from her vagina. The nurse concludes that: a. The placenta has separated. b. A cervical tear occurred during the birth. c. The woman is beginning to hemorrhage. d. Clots have formed in the upper uterine segment.

ANS: A Placental separation is indicated by a firmly contracting uterus, a change in the uterus from a discoid to a globular ovoid shape, a sudden gush of dark red blood from the introitus, an apparent lengthening of the umbilical cord, and a finding of vaginal fullness. Cervical tears that do not extend to the vagina result in minimal blood loss. Signs of hemorrhage are a boggy uterus, bright red vaginal bleeding, alterations in vital signs, pallor, lightheadedness, restlessness, decreased urinary output, and alteration in the level of consciousness. If clots have formed in the upper uterine segment, the nurse would expect to find the uterus boggy and displaced to the side.

17. Nurses should be aware of the differences experience can make in labor pain such as: a. Sensory pain for nulliparous women often is greater than for multiparous women during early labor. b. Affective pain for nulliparous women usually is less than for multiparous women throughout the first stage of labor. c. Women with a history of substance abuse experience more pain during labor. d. Multiparous women have more fatigue from labor and therefore experience more pain.

ANS: A Sensory pain is greater for nulliparous women because their reproductive tract structures are less supple. Affective pain is greater for nulliparous women during the first stage but decreases for both nulliparous and multiparous during the second stage. Women with a history of substance abuse experience the same amount of pain as those without such a history. Nulliparous women have longer labors and therefore experience more fatigue.

25. In assessing a woman for pain and discomfort management during labor, a nurse most likely would: a. Have the woman use a visual analog scale (VAS) to determine her level of pain. b. Note drowsiness as a sign that the medications were working. c. Interpret a womans fist clenching as an indication that she is angry at her male partner and the physician. d. Evaluate the womans skin turgor to see whether she needs a gentle oil massage.

ANS: A The VAS is a means of adding the womans assessment of her pain to the nurses observations. Drowsiness is a side effect of medications, not usually (sedatives aside) a sign of effectiveness. The fist clenching likely is a sign of apprehension that may need attention. Skin turgor, along with the moistness of the membranes and the concentration of the urine, is a sign that helps the nurse evaluate hydration.

22. When using intermittent auscultation (IA) to assess uterine activity, the nurse should be cognizant that: a. The examiners hand should be placed over the fundus before, during, and after contractions. b. The frequency and duration of contractions is measured in seconds for consistency. c. Contraction intensity is given a judgment number of 1 to 7 by the nurse and client together. d. The resting tone between contractions is described as either placid or turbulent.

ANS: A The assessment is done by palpation duration, frequency, intensity, and resting tone must be assessed. The duration of contractions is measured in seconds the frequency is measured in minutes. The intensity of contractions usually is described as mild, moderate, or strong. The resting tone usually is characterized as soft or relaxed.

14. The most critical nursing action in caring for the newborn immediately after birth is: a. Keeping the newborns airway clear. b. Fostering parent-newborn attachment. c. Drying the newborn and wrapping the infant in a blanket. d. Administering eye drops and vitamin K.

ANS: A The care given immediately after the birth focuses on assessing and stabilizing the newborn. Although fostering parent-infant attachment is an important task for the nurse, it is not the most critical nursing action in caring for the newborn immediately after birth. The nursing activities would be (in order of importance) to maintain a patent airway, support respiratory effort, and prevent cold stress by drying the newborn and covering the infant with a warmed blanket or placing the newborn under a radiant warmer. After the newborn has been stabilized, the nurse assesses the newborns physical condition, weighs and measures the newborn, administers prophylactic eye ointment and a vitamin K injection, affixes an identification bracelet, wraps the newborn in warm blankets, and then gives the infant to the partner or mother when he or she is ready.

34. The nurse knows that proper placement of the tocotransducer for electronic fetal monitoring is located: a. Over the uterine fundus. c. Inside the uterus. b. On the fetal scalp. d. Over the mothers lower abdomen.

ANS: A The tocotransducer monitors uterine activity and should be placed over the fundus, where the most intensive uterine contractions occur. The tocotransducer is for external use.

7. When assessing a woman in the first stage of labor, the nurse recognizes that the most conclusive sign that uterine contractions are effective would be: a. Dilation of the cervix. c. Rupture of the amniotic membranes. b. Descent of the fetus. d. Increase in bloody show.

ANS: A The vaginal examination reveals whether the woman is in true labor. Cervical change, especially dilation, in the presence of adequate labor indicates that the woman is in true labor. Descent of the fetus, or engagement, may occur before labor. Rupture of membranes may occur with or without the presence of labor. Bloody show may indicate slow, progressive cervical change (e.g., effacement) in both true and false labor.

31. In assisting with the two factors that have an effect on fetal status (i.e., pushing and positioning), nurses should: a. Encourage the womans cooperation in avoiding the supine position. b. Advise the woman to avoid the semi-Fowler position. c. Encourage the woman to hold her breath and tighten her abdominal muscles to produce a vaginal response. d. Instruct the woman to open her mouth and close her glottis, letting air escape after the push.

ANS: A The woman should maintain a side-lying position. The semi-Fowler position is the recommended side-lying position with a lateral tilt to the uterus. The Valsalva maneuver, which encourages the woman to hold her breath and tighten her abdominal muscles, should be avoided. Both the mouth and glottis should be open, letting air escape during the push.

13. To help clients manage discomfort and pain during labor, nurses should be aware that: a. The predominant pain of the first stage of labor is the visceral pain located in the lower portion of the abdomen. b. Referred pain is the extreme discomfort between contractions. c. The somatic pain of the second stage of labor is more generalized and related to fatigue. d. Pain during the third stage is a somewhat milder version of the second stage.

ANS: A This pain comes from cervical changes, distention of the lower uterine segment, and uterine ischemia. Referred pain occurs when the pain that originates in the uterus radiates to the abdominal wall, lumbosacral area of the back, iliac crests, and gluteal area. Second-stage labor pain is intense, sharp, burning, and localized. Third- stage labor pain is similar to that of the first stage.

12. When managing the care of a woman in the second stage of labor, the nurse uses various measures to enhance the progress of fetal descent. These measures include: a. Encouraging the woman to try various upright positions, including squatting and standing. b. Telling the woman to start pushing as soon as her cervix is fully dilated. c. Continuing an epidural anesthetic so pain is reduced and the woman can relax. d. Coaching the woman to use sustained, 10- to 15-second, closed-glottis bearing-down efforts with each contraction.

ANS: A Upright positions and squatting both may enhance the progress of fetal descent. Many factors dictate when a woman will begin pushing. Complete cervical dilation is necessary, but it is only one factor. If the fetal head is still in a higher pelvic station, the physician or midwife may allow the woman to labor down (allowing more time for fetal descent, thereby reducing the amount of pushing needed) if the woman is able. The epidural may mask the sensations and muscle control needed for the woman to push effectively. Closed glottic breathing may trigger the Valsalva maneuver, which increases intrathoracic and cardiovascular pressures, reducing cardiac output and inhibiting perfusion of the uterus and placenta. In addition, holding the breath for longer than 5 to 7 seconds diminishes the perfusion of oxygen across the placenta and results in fetal hypoxia.

5. When planning care for a laboring woman whose membranes have ruptured, the nurse recognizes that the womans risk for _________________________ has increased. a. Intrauterine infection c. Precipitous labor b. Hemorrhage d. Supine hypotension

ANS: A When the membranes rupture, microorganisms from the vagina can ascend into the amniotic sac and cause chorioamnionitis and placentitis. Rupture of membranes (ROM) is not associated with fetal or maternal bleeding. Although ROM may increase the intensity of contractions and facilitate active labor, it does not result in precipitous labor. ROM has no correlation with supine hypotension.

36. While developing an intrapartum care plan for the patient in early labor, it is important that the nurse recognize that psychosocial factors may influence a womans experience of pain. These include (Select all that apply): a. Culture. b. Anxiety and fear. c. Previous experiences with pain. d. Intervention of caregivers. e. Support systems.

ANS: A, B, C, E Culture: a womans sociocultural roots influence how she perceives, interprets, and responds to pain during childbirth. Some cultures encourage loud and vigorous expressions of pain, whereas others value self-control. The nurse should avoid praising some behaviors (stoicism) while belittling others (noisy expression). Anxiety and fear: extreme anxiety and fear magnify sensitivity to pain and impair a womans ability to tolerate it. Anxiety and fear increase muscle tension in the pelvic area, which counters the expulsive forces of uterine contractions and pushing efforts. Previous experiences with pain: fear and withdrawal are a natural response to pain during labor. Learning about these normal sensations ahead of time helps a woman suppress her natural reactions of fear regarding the impending birth. If a woman previously had a long and difficult labor, she is likely to be anxious. She may also have learned ways to cope and may use these skills to adapt to the present labor experience. Support systems: an anxious partner is less able to provide help and support to a woman during labor. A womans family and friends can be an important source of support if they convey realistic and positive information about labor and delivery. Although the intervention of caregivers may be necessary for the well-being of the woman and her fetus, some interventions add discomfort to the natural pain of labor (i.e., fetal monitor straps, intravenous lines).

35. The class of drugs known as opioid analgesics (butorphanol, nalbuphine) is not suitable for administration to women with known opioid dependence. The antagonistic activity could precipitate withdrawal symptoms (abstinence syndrome) in both mothers and newborns. Signs of opioid/narcotic withdrawal in the mother would include (Select all that apply): a. Yawning, runny nose. b. Increase in appetite. c. Chills and hot flashes. d. Constipation. e. Irritability, restlessness.

ANS: A, C, E The woman experiencing maternal opioid withdrawal syndrome will exhibit yawning, runny nose, sneezing, anorexia, chills or hot flashes, vomiting, diarrhea, abdominal pain, irritability, restlessness, muscle spasms, weakness, and drowsiness. It is important for the nurse to assess both mother and baby and to plan care accordingly.

Explain the fetal heart monitor to spouse: b. The top line graphs the babys heart rate. Generally the heart rate is between 110 and 160. The heart rate will fluctuate in response to what is happening during labor. c. The top line graphs the babys heart rate, and the bottom line lets me know how strong the contractions are. d. Your doctor will explain all of that later.

ANS: B The top line graphs the babys heart rate. Generally the heart rate is between 110 and 160. The heart rate will fluctuate in response to what is happening during labor educates the partner about fetal monitoring and provides support and information to alleviate his fears. Dont worry about that machine

4. What is an expected characteristic of amniotic fluid? a. Deep yellow color b. Pale, straw color with small white particles c. Acidic result on a Nitrazine test d. Absence of ferning

ANS: B Amniotic fluid normally is a pale, straw-colored fluid that may contain white flecks of vernix. Yellow-stained fluid may indicate fetal hypoxia up to 36 hours before rupture of membranes, fetal hemolytic disease, or intrauterine infection. Amniotic fluid produces an alkaline result on a Nitrazine test. The presence of ferning is a positive indication of amniotic fluid.

16. With regard to a pregnant womans anxiety and pain experience, nurses should be aware that: a. Even mild anxiety must be treated. b. Severe anxiety increases tension, which increases pain, which in turn increases fear and anxiety, and so on. c. Anxiety may increase the perception of pain, but it does not affect the mechanism of labor. d. Women who have had a painful labor will have learned from the experience and have less anxiety the second time because of increased familiarity.

ANS: B Anxiety and pain reinforce each other in a negative cycle. Mild anxiety is normal for a woman in labor and likely needs no special treatment other than the standard reassurances. Anxiety increases muscle tension and ultimately can build sufficiently to slow the progress of labor. Unfortunately, an anxious, painful first labor is likely to carry over, through expectations and memories, into an anxious and painful experience in the second pregnancy.

26. After change-of-shift report the nurse assumes care of a multiparous client in labor. The woman is complaining of pain that radiates to her abdominal wall, lower back, and buttocks and down her thighs. Before implementing a plan of care, the nurse should understand that this type of pain is: a. Visceral. c. Somatic. b. Referred. d. Afterpain.

ANS: B As labor progresses the woman often experiences referred pain. This occurs when pain that originates in the uterus radiates to the abdominal wall, the lumbosacral area of the back, the gluteal area, and thighs. The woman usually has pain only during a contraction and is free from pain between contractions. Visceral pain is that which predominates in the first stage of labor. This pain originates from cervical changes, distention of the lower uterine segment, and uterine ischemia. Visceral pain is located over the lower portion of the abdomen. Somatic pain is described as intense, sharp, burning, and well localized. This results from stretching of the perineal tissues and the pelvic floor. This occurs during the second stage of labor. Pain experienced during the third stage of labor or afterward during the early postpartum period is uterine. This pain is very similar to that experienced in the first stage of labor.

27. It is important for the nurse to develop a realistic birth plan with the pregnant woman in her care. The nurse can explain that a major advantage of nonpharmacologic pain management is: a. Greater and more complete pain relief is possible. b. No side effects or risks to the fetus are involved. c. The woman remains fully alert at all times. d. A more rapid labor is likely.

ANS: B Because nonpharmacologic pain management does not include analgesics, adjunct drugs, or anesthesia, it is harmless to the mother and the fetus. There is less pain relief with nonpharmacologic pain management during childbirth. The womans alertness is not altered by medication however, the increase in pain will decrease alertness. Pain management may or may not alter the length of labor. At times when pain is decreased, the mother relaxes and labor progresses at a quicker pace.

20. For the labor nurse, care of the expectant mother begins with any or all of these situations, with the exception of: a. The onset of progressive, regular contractions. b. The bloody, or pink, show. c. The spontaneous rupture of membranes. d. Formulation of the womans plan of care for labor.

ANS: D Labor care begins when progressive, regular contractions begin the blood-tinged mucoid vaginal discharge appears or fluid is discharged from the vagina. The woman and nurse can formulate their plan of care before labor or during treatment.

21. With regard to systemic analgesics administered during labor, nurses should be aware that: a. Systemic analgesics cross the maternal blood-brain barrier as easily as they do the fetal blood-brain barrier. b. Effects on the fetus and newborn can include decreased alertness and delayed sucking. c. Intramuscular administration (IM) is preferred over intravenous (IV) administration. d. IV patient-controlled analgesia (PCA) results in increased use of an analgesic.

ANS: B Effects depend on the specific drug given, the dosage, and the timing. Systemic analgesics cross the fetal blood-brain barrier more readily than the maternal blood-brain barrier. IV administration is preferred over IM administration because the drug acts faster and more predictably. PCA results in decreased use of an analgesic.

23. With regard to spinal and epidural (block) anesthesia, nurses should know that: a. This type of anesthesia is commonly used for cesarean births but is not suitable for vaginal births. b. A high incidence of after-birth headache is seen with spinal blocks. c. Epidural blocks allow the woman to move freely. d. Spinal and epidural blocks are never used together.

ANS: B Headaches may be prevented or mitigated to some degree by a number of methods. Spinal blocks may be used for vaginal births, but the woman must be assisted through labor. Epidural blocks limit the womans ability to move freely. Combined use of spinal and epidural blocks is becoming increasingly popular.

19. A woman who is 39 weeks pregnant expresses fear about her impending labor and how she will manage. The nurses best response is: a. Dont worry about it. Youll do fine. b. Its normal to be anxious about labor. Lets discuss what makes you afraid. c. Labor is scary to think about, but the actual experience isnt. d. You can have an epidural. You wont feel anything.

ANS: B Its normal to be anxious about labor. Lets discuss what makes you afraid allows the woman to share her concerns with the nurse and is a therapeutic communication tool. Dont worry about it. Youll do fine negates the womans fears and is not therapeutic. Labor is scary to think about, but the actual experience isnt negates the womans fears and offers a false sense of security. It is not true that every woman may have an epidural. A number of criteria must be met for use of an epidural. Furthermore, many women still experience the feeling of pressure with an epidural.

28. What correctly matches the type of deceleration with its likely cause? a. Early decelerationumbilical cord compression b. Late decelerationuteroplacental inefficiency c. Variable decelerationhead compression d. Prolonged decelerationcause unknown

ANS: B Late deceleration is caused by uteroplacental inefficiency. Early deceleration is caused by head compression. Variable deceleration is caused by umbilical cord compression. Prolonged deceleration has a variety of either benign or critical causes.

11. The role of the nurse with regard to informed consent is to: a. Inform the client about the procedure and have her sign the consent form. b. Act as a client advocate and help clarify the procedure and the options. c. Call the physician to see the client. d. Witness the signing of the consent form.

ANS: B Nurses play a part in the informed consent process by clarifying and describing procedures or by acting as the womans advocate and asking the primary health care provider for further explanations. The physician is responsible for informing the woman of her options, explaining the procedure, and advising the client about potential risk factors. The physician must be present to explain the procedure to the client. However, the nurses responsibilities go further than simply asking the physician to see the client. The nurse may witness the signing of the consent form. However, depending on the states guidelines, the womans husband or another hospital health care employee may sign as witness.

16. The nurse expects to administer an oxytocic (e.g., Pitocin, Methergine) to a woman after expulsion of her placenta to: a. Relieve pain. c. Prevent infection. b. Stimulate uterine contraction. d. Facilitate rest and relaxation.

ANS: B Oxytocics stimulate uterine contractions, which reduce blood loss after the third stage of labor. Oxytocics are not used to treat pain or prevent infection. They cause the uterus to contract, which reduces blood loss. Oxytocics do not facilitate rest and relaxation.

15. What three measures should the nurse implement to provide intrauterine resuscitation? Select the response that best indicates the priority of actions that should be taken. a. Call the provider, reposition the mother, and perform a vaginal examination. b. Reposition the mother, increase intravenous (IV) fluid, and provide oxygen via face mask. c. Administer oxygen to the mother, increase IV fluid, and notify the care provider. d. Perform a vaginal examination, reposition the mother, and provide oxygen via face mask.

ANS: B Repositioning the mother, increasing intravenous (IV) fluid, and providing oxygen via face mask are correct nursing actions for intrauterine resuscitation. The nurse should initiate intrauterine resuscitation in an ABC manner, similar to basic life support. The first priority is to open the maternal and fetal vascular systems by repositioning the mother for improved perfusion. The second priority is to increase blood volume by increasing the IV fluid. The third priority is to optimize oxygenation of the circulatory volume by providing oxygen via face mask. If these interventions do not resolve the fetal heart rate issue quickly, the primary provider should be notified immediately.

12. A first-time mother is concerned about the type of medications she will receive during labor. She is in a fair amount of pain and is nauseous. In addition, she appears to be very anxious. You explain that opioid analgesics often are used with sedatives because: a. The two together work the best for you and your baby. b. Sedatives help the opioid work better, and they also will assist you to relax and relieve your nausea. c. They work better together so you can sleep until you have the baby. d. This is what the doctor has ordered for you.

ANS: B Sedatives can be used to reduce the nausea and vomiting that often accompany opioid use. In addition, some ataractics reduce anxiety and apprehension and potentiate the opioid analgesic affects. A potentiator may cause the two drugs to work together more effectively, but it does not ensure maternal or fetal complications will not occur. Sedation may be a related effect of some ataractics, but it is not the goal. Furthermore, a woman is unlikely to be able to sleep through transitional labor and birth. This is what the doctor has ordered for you may be true, but it is not an acceptable comment for the nurse to make.

6. The nurse should be aware that an effective plan to achieve adequate pain relief without maternal risk is most effective if: a. The mother gives birth without any analgesic or anesthetic. b. The mother and familys priorities and preferences are incorporated into the plan. c. The primary health care provider decides the best pain relief for the mother and family. d. The nurse informs the family of all alternative methods of pain relief available in the hospital setting.

ANS: B The assessment of the woman, her fetus, and her labor is a joint effort of the nurse and the primary health care providers, who consult with the woman about their findings and recommendations. The needs of each woman are different, and many factors must be considered before a decision is made whether pharmacologic methods, nonpharmacologic methods, or a combination of the two will be used to manage labor pain.

25. During labor a fetus with an average heart rate of 135 beats/min over a 10-minute period would be considered to have: a. Bradycardia. c. Tachycardia. b. A normal baseline heart rate. d. Hypoxia.

ANS: B The baseline heart rate is measured over 10 minutes a normal range is 110 to 160 beats/min. Bradycardia is a fetal heart rate (FHR) below 110 beats/min for 10 minutes or longer. Tachycardia is an FHR over 160 beats/min for 10 minutes or longer. Hypoxia is an inadequate supply of oxygen

13. Through vaginal examination the nurse determines that a woman is 4 cm dilated, and the external fetal monitor shows uterine contractions every 3.5 to 4 minutes. The nurse would report this as: a. First stage, latent phase. c. First stage, transition phase. b. First stage, active phase. d. Second stage, latent phase.

ANS: B The first stage, active phase of maternal progress indicates that the woman is in the active phase of the first stage of labor. During the latent phase of the first stage of labor, the expected maternal progress would be 0 to 3 cm dilation with contractions every 5 to 30 minutes. During the transition phase of the first stage of labor, the expected maternal progress is 8 to 10 cm dilation with contractions every 2 to 3 minutes. During the latent phase of the second stage of labor, the woman is completely dilated and experiences a restful period of laboring down.

11. Which fetal heart rate (FHR) finding would concern the nurse during labor? a. Accelerations with fetal movement c. An average FHR of 126 beats/min b. Early decelerations d. Late decelerations

ANS: D Late decelerations are caused by uteroplacental insufficiency and are associated with fetal hypoxemia. They are considered ominous if persistent and uncorrected. Accelerations in the FHR are an indication of fetal well- being. Early decelerations in the FHR are associated with head compression as the fetus descends into the maternal pelvic outlet they generally are not a concern during normal labor.

33. To assist the woman after delivery of the infant, the nurse knows that the blood patch is used after spinal anesthesia to relieve: a. Hypotension. c. Neonatal respiratory depression. b. Headache. d. Loss of movement.

ANS: B The subarachnoid block may cause a postspinal headache resulting from loss of cerebrospinal fluid from the puncture in the dura. When blood is injected into the epidural space in the area of the dural puncture, it forms a seal over the hole to stop leaking of cerebrospinal fluid. Hypotension is prevented by increasing fluid volume before the procedure. Neonatal respiratory depression is not an expected outcome with spinal anesthesia. Loss of movement is an expected outcome of spinal anesthesia.

35. Why is continuous electronic fetal monitoring usually used when oxytocin is administered? a. The mother may become hypotensive. b. Uteroplacental exchange may be compromised. c. Maternal fluid volume deficit may occur. d. Fetal chemoreceptors are stimulated.

ANS: B The uterus may contract more firmly, and the resting tone may be increased with oxytocin use. This response reduces entrance of freshly oxygenated maternal blood into the intervillous spaces, thus depleting fetal oxygen reserves. Hypotension is not a common side effect of oxytocin. All laboring women are at risk for fluid volume deficit oxytocin administration does not increase the risk. Oxytocin affects the uterine muscles.

9. If an opioid antagonist is administered to a laboring woman, she should be told that: a. Her pain will decrease. b. Her pain will return. c. She will feel less anxious. d. She will no longer feel the urge to push.

ANS: B The woman should be told that the pain that was relieved by the opioid analgesic will return with administration of the opioid antagonist. Opioid antagonists, such as Narcan, promptly reverse the central nervous system (CNS) depressant effects of opioids. In addition, the antagonist counters the effect of the stress-induced levels of endorphins. An opioid antagonist is especially valuable if labor is more rapid than expected and birth is anticipated when the opioid is at its peak effect.

5. The nurse providing care for the laboring woman realizes that variable fetal heart rate (FHR) decelerations are caused by: a. Altered fetal cerebral blood flow. c. Uteroplacental insufficiency. b. Umbilical cord compression. d. Fetal hypoxemia.

ANS: B Variable decelerations can occur any time during the uterine contracting phase and are caused by compression of the umbilical cord. Altered fetal cerebral blood flow would result in early decelerations in the FHR. Uteroplacental insufficiency would result in late decelerations in the FHR. Fetal hypoxemia would result in tachycardia initially and then bradycardia if hypoxia continues.

MULTIPLE RESPONSE 34. Maternal hypotension is a potential side effect of regional anesthesia and analgesia. What nursing interventions could you use to raise the clients blood pressure (Select all that apply)? a. Place the woman in a supine position. b. Place the woman in a lateral position. c. Increase intravenous (IV) fluids. d. Administer oxygen. e. Perform a vaginal examination.

ANS: B, C, D Nursing interventions for maternal hypotension arising from analgesia or anesthesia include turning the woman to a lateral position, increasing IV fluids, administering oxygen via face mask, elevating the womans legs, notifying the physician, administering an IV vasopressor, and monitoring the maternal and fetal status at least every 5 minutes until these are stable. Placing the client in a supine position would cause venous compression, thereby limiting blood flow to and oxygenation of the placenta and fetus. A sterile vaginal examination has no bearing on maternal blood pressure.

MULTIPLE RESPONSE 37. A tiered system of categorizing FHR has been recommended by regulatory agencies. Nurses, midwives, and physicians who care for women in labor must have a working knowledge of fetal monitoring standards and understand the significance of each category. These categories include (Select all that apply): a. Reassuring. b. Category I. c. Category II. d. Nonreassuring. e. Category III.

ANS: B, C, E The three tiered system of FHR tracings include Category I, II, and III. Category I is a normal tracing requiring no action. Category II FHR tracings are indeterminate. This category includes tracings that do not meet Category I or III criteria. Category III tracings are abnormal and require immediate intervention.

22. It is paramount for the obstetric nurse to understand the regulatory procedures and criteria for admitting a woman to the hospital labor unit. Which guideline is an important legal requirement of maternity care? a. The patient is not considered to be in true labor (according to the Emergency Medical Treatment and Active Labor Act [EMTALA]) until a qualified health care provider says she is. b. The woman can have only her male partner or predesignated doula with her at assessment. c. The patients weight gain is calculated to determine whether she is at greater risk for cephalopelvic disproportion (CPD) and cesarean birth. d. The nurse may exchange information about the patient with family members.

ANS: C According to EMTALA, a woman is entitled to active labor care and is presumed to be in true labor until a qualified health care provider certifies otherwise. A woman can have anyone she wishes present for her support. The risk for CPD is especially great for petite women or those who have gained 16 kg or more. All patients should have their weight and BMI calculated on admission. This is part of standard nursing care on a maternity unit and not a regulatory concern. According to the Health Insurance Portability and Accountability Act (HIPAA), the patient must give consent for others to receive any information related to her condition.

4. A laboring woman received an opioid agonist (meperidine) intravenously 90 minutes before she gave birth. Which medication should be available to reduce the postnatal effects of Demerol on the neonate? a. Fentanyl (Sublimaze) c. Naloxone (Narcan) b. Promethazine (Phenergan) d. Nalbuphine (Nubain)

ANS: C An opioid antagonist can be given to the newborn as one part of the treatment for neonatal narcosis, which is a state of central nervous system (CNS) depression in the newborn produced by an opioid. Opioid antagonists such as naloxone (Narcan) can promptly reverse the CNS depressant effects, especially respiratory depression. Fentanyl, promethazine, and nalbuphine do not act as opioid antagonists to reduce the postnatal effects of Demerol on the neonate. Although meperidine (Demerol) is a low-cost medication and readily available, the use of Demerol in labor has been controversial because of its effects on the neonate.

10. A nulliparous woman who has just begun the second stage of her labor would most likely: a. Experience a strong urge to bear down. b. Show perineal bulging. c. Feel tired yet relieved that the worst is over. d. Show an increase in bright red bloody show.

ANS: C Common maternal behaviors during the latent phase of the second stage of labor include feeling a sense of accomplishment and optimism because the worst is over. During the latent phase of the second stage of labor, the urge to bear down often is absent or only slight during the acme of contractions. Perineal bulging occurs during the transition phase of the second stage of labor, not at the beginning of the second stage. An increase in bright red bloody show occurs during the descent phase of the second stage of labor.

11. The nurse knows that the second stage of labor, the descent phase, has begun when: a. The amniotic membranes rupture. b. The cervix cannot be felt during a vaginal examination. c. The woman experiences a strong urge to bear down. d. The presenting part is below the ischial spines.

ANS: C During the descent phase of the second stage of labor, the woman may experience an increase in the urge to bear down. Rupture of membranes has no significance in determining the stage of labor. The second stage of labor begins with full cervical dilation. Many women may have an urge to bear down when the presenting part is below the level of the ischial spines. This can occur during the first stage of labor, as early as 5-cm dilation.

1. Fetal bradycardia is most common during: a. Intraamniotic infection. b. Fetal anemia. c. Prolonged umbilical cord compression. d. Tocolytic treatment using terbutaline.

ANS: C Fetal bradycardia can be considered a later sign of fetal hypoxia and is known to occur before fetal death. Bradycardia can result from placental transfer of drugs, prolonged compression of the umbilical cord, maternal hypothermia, and maternal hypotension. Intraamniotic infection, fetal anemia, and tocolytic treatment using terbutaline would most likely result in fetal tachycardia.

15. Nurses with an understanding of cultural differences regarding likely reactions to pain may be better able to help clients. Nurses should know that _____ women may be stoic until late in labor, when they may become vocal and request pain relief. a. Chinese c. Hispanic b. Arab or Middle Eastern d. African-American

ANS: C Hispanic women may be stoic early and more vocal and ready for medications later. Chinese women may not show reactions to pain. Medical interventions must be offered more than once. Arab or Middle Eastern women may be vocal in response to labor pain from the start. They may prefer pain medications. African-American women may express pain openly use of medications for pain is more likely to vary with the individual.

8. The nurse caring for the woman in labor should understand that maternal hypotension can result in: a. Early decelerations. c. Uteroplacental insufficiency. b. Fetal dysrhythmias. d. Spontaneous rupture of membranes.

ANS: C Low maternal blood pressure reduces placental blood flow during uterine contractions and results in fetal hypoxemia. Maternal hypotension is not associated with early decelerations, fetal dysrhythmias, or spontaneous rupture of membranes.

29. The nurse caring for a woman in labor understands that prolonged decelerations: a. Are a continuing pattern of benign decelerations that do not require intervention. b. Constitute a baseline change when they last longer than 5 minutes. c. Usually are isolated events that end spontaneously. d. Require the usual fetal monitoring by the nurse.

ANS: C Prolonged decelerations usually are isolated events that end spontaneously. However, in certain combinations with late and/or variable decelerations, they are a danger sign that requires the nurse to notify the physician or midwife immediately. A deceleration that lasts longer than 10 minutes constitutes a baseline change.

1. The nurse recognizes that a woman is in true labor when she states: a. I passed some thick, pink mucus when I urinated this morning. b. My bag of waters just broke. c. The contractions in my uterus are getting stronger and closer together. d. My baby dropped, and I have to urinate more frequently now.

ANS: C Regular, strong contractions with the presence of cervical change indicate that the woman is experiencing true labor. Loss of the mucous plug (operculum) often occurs during the first stage of labor or before the onset of labor, but it is not the indicator of true labor. Spontaneous rupture of membranes often occurs during the first stage of labor, but it is not the indicator of true labor. The presenting part of the fetus typically becomes engaged in the pelvis at the onset of labor, but this is not the indicator of true labor.

20. According to standard professional thinking, nurses should auscultate the fetal heart rate (FHR): a. Every 15 minutes in the active phase of the first stage of labor in the absence of risk factors. b. Every 20 minutes in the second stage, regardless of whether risk factors are present. c. Before and after ambulation and rupture of membranes. d. More often in a womans first pregnancy.

ANS: C The FHR should be auscultated before and after administration of medications and induction of anesthesia. In the active phase of the first stage of labor, the FHR should be auscultated every 30 minutes if no risk factors are involved with risk factors it should be auscultated every 15 minutes. In the second stage of labor the FHR should be auscultated every 15 minutes if no risk factors are involved with risk factors it should be auscultated every 5 minutes. The fetus of a first-time mother is automatically at greater risk.

5. A woman in labor has just received an epidural block. The most important nursing intervention is to: a. Limit parenteral fluids. b. Monitor the fetus for possible tachycardia. c. Monitor the maternal blood pressure for possible hypotension. d. Monitor the maternal pulse for possible bradycardia.

ANS: C The most important nursing intervention for a woman who has received an epidural block is to monitor the maternal blood pressure frequently for signs of hypotension. Intravenous fluids are increased for a woman receiving an epidural, to prevent hypotension. The nurse observes for signs of fetal bradycardia. The nurse monitors for signs of maternal tachycardia secondary to hypotension.

32. The uterine contractions of a woman early in the active phase of labor are assessed by an internal uterine pressure catheter (IUPC). The nurse notes that the intrauterine pressure at the peak of the contraction ranges from 65 to 70 mm Hg and the resting tone range is 6 to 10 mm Hg. The uterine contractions occur every 3 to 4 minutes and last an average of 55 to 60 seconds. On the basis of this information, the nurse should: a. Notify the womans primary health care provider immediately. b. Prepare to administer an oxytocic to stimulate uterine activity. c. Document the findings because they reflect the expected contraction pattern for the active phase of labor. d. Prepare the woman for the onset of the second stage of labor.

ANS: C The nurse is responsible for monitoring the uterine contractions to ascertain whether they are powerful and frequent enough to accomplish the work of expelling the fetus and the placenta. In addition, the nurse would document these findings in the clients medical record. This labor pattern indicates that the client is in the active phase of the first stage of labor. Nothing indicates a need to notify the primary care provider at this time. Oxytocin augmentation is not needed for this labor pattern this contraction pattern indicates adequate active labor. Her contractions eventually will become stronger, last longer, and come closer together during the transition phase of the first stage of labor. The transition phase precedes the second stage of labor, or delivery of the fetus.

6. Which action is correct when palpation is used to assess the characteristics and pattern of uterine contractions? a. Place the hand on the abdomen below the umbilicus and palpate uterine tone with the fingertips. b. Determine the frequency by timing from the end of one contraction to the end of the next contraction. c. Evaluate the intensity by pressing the fingertips into the uterine fundus. d. Assess uterine contractions every 30 minutes throughout the first stage of labor.

ANS: C The nurse or primary care provider may assess uterine activity by palpating the fundal section of the uterus using the fingertips. Many women may experience labor pain in the lower segment of the uterus that may be unrelated to the firmness of the contraction detectable in the uterine fundus. The frequency of uterine contractions is determined by palpating from the beginning of one contraction to the beginning of the next contraction. Assessment of uterine activity is performed in intervals based on the stage of labor. As labor progresses this assessment is performed more frequently.

10. A woman has requested an epidural for her pain. She is 5 cm dilated and 100% effaced. The baby is in a vertex position and is engaged. The nurse increases the womans intravenous fluid for a preprocedural bolus. She reviews her laboratory values and notes that the womans hemoglobin is 12 g/dL, hematocrit is 38%, platelets are 67,000, and white blood cells (WBCs) are 12,000/mm3 . Which factor would contraindicate an epidural for the woman? a. She is too far dilated. c. She has thrombocytopenia. b. She is anemic. d. She is septic.

ANS: C The platelet count indicates a coagulopathy, specifically, thrombocytopenia (low platelets), which is a contraindication to epidural analgesia/anesthesia. Typically epidural analgesia/anesthesia is used in the laboring woman when a regular labor pattern has been achieved, as evidenced by progressive cervical change. The laboratory values show that the womans hemoglobin and hematocrit are in the normal range and show a slight increase in the WBC count that is not uncommon in laboring women.

23. What is an advantage of external electronic fetal monitoring? a. The ultrasound transducer can accurately measure short-term variability and beat-to-beat changes in the fetal heart rate. b. The tocotransducer can measure and record the frequency, regularity, intensity, and approximate duration of uterine contractions (UCs). c. The tocotransducer is especially valuable for measuring uterine activity during the first stage of labor. d. Once correctly applied by the nurse, the transducer need not be repositioned even when the woman changes positions.

ANS: C The tocotransducer is especially valuable for measuring uterine activity during the first stage of labor, particularly when the membranes are intact. Short-term changes cannot be measured with this technology. The tocotransducer cannot measure and record the intensity of UCs. The transducer must be repositioned when the woman or fetus changes position.

24. A woman in labor is breathing into a mouthpiece just before the start of her regular contractions. As she inhales, a valve opens, and gas is released. She continues to inhale the gas slowly and deeply until the contraction starts to subside. When the inhalation stops, the valve closes. This procedure is: a. Not used much anymore. b. Likely to be used in the second stage of labor but not in the first stage. c. An application of nitrous oxide. d. A prelude to cesarean birth.

ANS: C This is an application of nitrous oxide, which could be used in either the first or second stage of labor (or both) as part of the preparation for a vaginal birth. Nitrous oxide is self-administered and found to be very helpful.

2. The nurse teaches a pregnant woman about the characteristics of true labor contractions. The nurse evaluates the womans understanding of the instructions when she states, True labor contractions will: a. Subside when I walk around. b. Cause discomfort over the top of my uterus. c. Continue and get stronger even if I relax and take a shower. d. Remain irregular but become stronger.

ANS: C True labor contractions occur regularly, becoming stronger, lasting longer, and occurring closer together. They may become intense during walking and continue despite comfort measures. Typically true labor contractions are felt in the lower back, radiating to the lower portion of the abdomen. During false labor, contractions tend to be irregular and felt in the abdomen above the navel. Typically the contractions often stop with walking or a change of position.

6. The nurse providing care for the laboring woman should understand that late fetal heart rate (FHR) decelerations are the result of: a. Altered cerebral blood flow. c. Uteroplacental insufficiency. b. Umbilical cord compression. d. Meconium fluid.

ANS: C Uteroplacental insufficiency would result in late decelerations in the FHR. Altered fetal cerebral blood flow would result in early decelerations in the FHR. Umbilical cord compression would result in variable decelerations in the FHR. Meconium-stained fluid may or may not produce changes in the fetal heart rate, depending on the gestational age of the fetus and whether other causative factors associated with fetal distress are present.

38. The baseline fetal heart rate (FHR) is the average rate during a 10-minute segment. Changes in FHR are categorized as periodic or episodic. These patterns include both accelerations and decelerations. The labor nurse is evaluating the patients most recent 10-minute segment on the monitor strip and notes a late deceleration. This is likely to be caused by which physiologic alteration (Select all that apply)? a. Spontaneous fetal movement b. Compression of the fetal head c. Placental abruption d. Cord around the babys neck e. Maternal supine hypotension

ANS: C, E Late decelerations are almost always caused by uteroplacental insufficiency. Insufficiency is caused by uterine tachysystole, maternal hypotension, epidural or spinal anesthesia, IUGR, intraamniotic infection, or placental abruption. Spontaneous fetal movement, vaginal examination, fetal scalp stimulation, fetal reaction to external sounds, uterine contractions, fundal pressure and abdominal palpation are all likely to cause accelerations of the FHR. Early decelerations are most often the result of fetal head compression and may be caused by uterine contractions, fundal pressure, vaginal examination, and placement of an internal electrode. A variable deceleration is likely caused by umbilical cord compression. This may happen when the umbilical cord is around the babys neck, arm, leg, or other body part or when there is a short cord, a knot in the cord, or a prolapsed cord.

30. A nurse may be called on to stimulate the fetal scalp: a. As part of fetal scalp blood sampling. b. In response to tocolysis. c. In preparation for fetal oxygen saturation monitoring. d. To elicit an acceleration in the fetal heart rate (FHR).

ANS: D The scalp can be stimulated using digital pressure during a vaginal examination. Fetal scalp blood sampling involves swabbing the scalp with disinfectant before a sample is collected. The nurse would stimulate the fetal scalp to elicit an acceleration of the FHR. Tocolysis is relaxation of the uterus. Fetal oxygen saturation monitoring involves the insertion of a sensor.

7. A woman in the active phase of the first stage of labor is using a shallow pattern of breathing, which is about twice the normal adult breathing rate. She starts to complain about feeling lightheaded and dizzy and states that her fingers are tingling. The nurse should: a. Notify the womans physician. b. Tell the woman to slow the pace of her breathing. c. Administer oxygen via a mask or nasal cannula. d. Help her breathe into a paper bag

ANS: D This woman is experiencing the side effects of hyperventilation, which include the symptoms of lightheadedness, dizziness, tingling of the fingers, or circumoral numbness. Having the woman breathe into a paper bag held tightly around her mouth and nose may eliminate respiratory alkalosis. This enables her to rebreathe carbon dioxide and replace the bicarbonate ion.

12. The most common cause of decreased variability in the fetal heart rate (FHR) that lasts 30 minutes or less is: a. Altered cerebral blood flow. c. Umbilical cord compression. b. Fetal hypoxemia. d. Fetal sleep cycles.

ANS: D A temporary decrease in variability can occur when the fetus is in a sleep state. These sleep states do not usually last longer than 30 minutes. Altered fetal cerebral blood flow would result in early decelerations in the FHR. Fetal hypoxemia would be evidenced by tachycardia initially and then bradycardia. A persistent decrease or loss of FHR variability may be seen. Umbilical cord compression would result in variable decelerations in the FHR.

32. The obstetric nurse is preparing the patient for an emergency cesarean birth, with no time to administer spinal anesthesia. The nurse is aware and prepared for the greatest risk of administering general anesthesia to the patient. This risk is: a. Respiratory depression. c. Inadequate muscle relaxation. b. Uterine relaxation. d. Aspiration of stomach contents.

ANS: D Aspiration of acidic gastric contents with possible airway obstruction is a potentially fatal complication of general anesthesia. Respirations can be altered during general anesthesia, and the anesthesiologist will take precautions to maintain proper oxygenation. Uterine relaxation can occur with some anesthesia however, this can be monitored and prevented. Inadequate muscle relaxation can be improved with medication.

3. When a nulliparous woman telephones the hospital to report that she is in labor, the nurse initially should: a. Tell the woman to stay home until her membranes rupture. b. Emphasize that food and fluid intake should stop. c. Arrange for the woman to come to the hospital for labor evaluation. d. Ask the woman to describe why she believes she is in labor.

ANS: D Assessment begins at the first contact with the woman, whether by telephone or in person. By asking the woman to describe her signs and symptoms, the nurse can begin the assessment and gather data. The amniotic membranes may or may not spontaneously rupture during labor. The client may be instructed to stay home until the uterine contractions become strong and regular. The nurse may want to discuss the appropriate oral intake for early labor such as light foods or clear liquids, depending on the preference of the client or her primary health care provider. Before instructing the woman to come to the hospital, the nurse should initiate the assessment during the telephone interview.

1. An 18-year-old pregnant woman, gravida 1, is admitted to the labor and birth unit with moderate contractions every 5 minutes that last 40 seconds. The woman states, My contractions are so strong that I dont know what to do with myself. The nurse should: a. Assess for fetal well-being. b. Encourage the woman to lie on her side. c. Disturb the woman as little as possible. d. Recognize that pain is personalized for each individual.

ANS: D Each womans pain during childbirth is unique and is influenced by a variety of physiologic, psychosocial, and environmental factors. A critical issue for the nurse is how support can make a difference in the pain of the woman during labor and birth. Assessing for fetal well-being includes no information that would indicate fetal distress or a logical reason to be overly concerned about the well-being of the fetus. The left lateral position is used to alleviate fetal distress, not maternal stress. The nurse has an obligation to provide physical, emotional, and psychosocial care and support to the laboring woman. This client clearly needs support.

18. In the current practice of childbirth preparation, emphasis is placed on: a. The Dick-Read (natural) childbirth method. b. The Lamaze (psychoprophylactic) method. c. The Bradley (husband-coached) method. d. Having expectant parents attend childbirth preparation in any or no specific method.

ANS: D Encouraging expectant parents to attend childbirth preparation class is most important because preparation increases a womans confidence and thus her ability to cope with labor and birth. Although still popular, the method format of classes is being replaced with other offerings such as Hypnobirthing and Birthing from Within.

24. When assessing the relative advantages and disadvantages of internal and external electronic fetal monitoring, nurses comprehend that both: a. Can be used when membranes are intact. b. Measure the frequency, duration, and intensity of uterine contractions. c. May need to rely on the woman to indicate when uterine activity (UA) is occurring. d. Can be used during the antepartum and intrapartum periods.

ANS: D External monitoring can be used in both periods internal monitoring can be used only in the intrapartum period. For internal monitoring the membranes must have ruptured, and the cervix must be sufficiently dilated. Internal monitoring measures the intensity of contractions external monitoring cannot do this. With external monitoring, the woman may need to alert the nurse that UA is occurring internal monitoring does not require this.

36. Increasing the infusion rate of nonadditive intravenous fluids can increase fetal oxygenation primarily by: a. Maintaining normal maternal temperature. b. Preventing normal maternal hypoglycemia. c. Increasing the oxygen-carrying capacity of the maternal blood. d. Expanding maternal blood volume.

ANS: D Filling the mothers vascular system makes more blood available to perfuse the placenta and may correct hypotension. Increasing fluid volume may alter the maternal temperature only if she is dehydrated. Most intravenous fluids for laboring women are isotonic and do not provide extra glucose. Oxygen-carrying capacity is increased by adding more red blood cells.

14. Which statement correctly describes the effects of various pain factors? a. Higher prostaglandin levels arising from dysmenorrhea can blunt the pain of childbirth. b. Upright positions in labor increase the pain factor because they cause greater fatigue. c. Women who move around trying different positions are experiencing more pain. d. Levels of pain-mitigating b-endorphins are higher during a spontaneous, natural childbirth.

ANS: D Higher endorphin levels help women tolerate pain and reduce anxiety and irritability. Higher prostaglandin levels correspond to more severe labor pains. Upright positions in labor usually result in improved comfort and less pain. Moving freely to find more comfortable positions is important for reducing pain and muscle tension.

21. When using intermittent auscultation (IA) for fetal heart rate, nurses should be aware that: a. They can be expected to cover only two or three clients when IA is the primary method of fetal assessment. b. The best course is to use the descriptive terms associated with electronic fetal monitoring (EFM) when documenting results. c. If the heartbeat cannot be found immediately, a shift must be made to EFM. d. Ultrasound can be used to find the fetal heartbeat and reassure the mother if initial difficulty was a factor.

ANS: D Locating fetal heartbeats often takes time. Mothers can be reassured verbally and by the ultrasound pictures if ultrasound is used to help locate the heartbeat. When used as the primary method of fetal assessment, auscultation requires a nurse-to-client ratio of one to one. Documentation should use only terms that can be numerically defined the usual visual descriptions of EFM are inappropriate.

17. As a perinatal nurse you realize that a fetal heart rate that is tachycardic, is bradycardic, or has late decelerations or loss of variability is nonreassuring and is associated with: a. Hypotension. c. Maternal drug use. b. Cord compression. d. Hypoxemia.

ANS: D Nonreassuring heart rate patterns are associated with fetal hypoxemia. Fetal bradycardia may be associated with maternal hypotension. Fetal variable decelerations are associated with cord compression. Maternal drug use is associated with fetal tachycardia.

10. While evaluating an external monitor tracing of a woman in active labor whose labor is being induced, the nurse notes that the fetal heart rate (FHR) begins to decelerate at the onset of several contractions and returns to baseline before each contraction ends. The nurse should: a. Change the womans position. b. Discontinue the oxytocin infusion. c. Insert an internal monitor. d. Document the finding in the clients record.

ANS: D The FHR indicates early decelerations, which are not an ominous sign and do not require any intervention. The nurse should simply document these findings.

8. The nurse who performs vaginal examinations to assess a womans progress in labor should: a. Perform an examination at least once every hour during the active phase of labor. b. Perform the examination with the woman in the supine position. c. Wear two clean gloves for each examination. d. Discuss the findings with the woman and her partner.

ANS: D The nurse should discuss the findings of the vaginal examination with the woman and her partner and report them to the primary care provider. A vaginal examination should be performed only when indicated by the status of the woman and her fetus. The woman should be positioned to avoid supine hypotension. The examiner should wear a sterile glove while performing a vaginal examination for a laboring woman.

9. A multiparous woman has been in labor for 8 hours. Her membranes have just ruptured. The nurses initial response would be to: a. Prepare the woman for imminent birth. b. Notify the womans primary health care provider. c. Document the characteristics of the fluid. d. Assess the fetal heart rate and pattern.

ANS: D The umbilical cord may prolapse when the membranes rupture. The fetal heart rate and pattern should be monitored closely for several minutes immediately after ROM to ascertain fetal well-being, and the findings should be documented. Rupture of membranes (ROM) may increase the intensity and frequency of the uterine contractions, but it does not indicate that birth is imminent. The nurse may notify the primary care provider after ROM occurs and fetal well-being and the response to ROM have been assessed. The nurses priority is to assess fetal well-being. The nurse should document the characteristics of the amniotic fluid, but the initial response is to assess fetal well-being and the response to ROM.

14. You are evaluating the fetal monitor tracing of your client, who is in active labor. Suddenly you see the fetal heart rate (FHR) drop from its baseline of 125 down to 80. You reposition the mother, provide oxygen, increase intravenous (IV) fluid, and perform a vaginal examination. The cervix has not changed. Five minutes have passed, and the fetal heart rate remains in the 80s. What additional nursing measures should you take? a. Scream for help. b. Insert a Foley catheter. c. Start Pitocin. d. Notify the care provider immediately.

ANS: D To relieve an FHR deceleration, the nurse can reposition the mother, increase IV fluid, and provide oxygen. If oxytocin is infusing, it should be discontinued. If the FHR does not resolve, the primary care provider should be notified immediately. Inserting a Foley catheter is an inappropriate nursing action. If the FHR were to continue in a nonreassuring pattern, a cesarean section could be warranted, which would require a Foley catheter. However, the physician must make that determination. Pitocin may place additional stress on the fetus.

20. Maternity nurses often have to answer questions about the many, sometimes unusual ways people have tried to make the birthing experience more comfortable. For instance, nurses should be aware that: a. Music supplied by the support person has to be discouraged because it could disturb others or upset the hospital routine. b. Women in labor can benefit from sitting in a bathtub, but they must limit immersion to no longer than 15 minutes at a time. c. Effleurage is permissible, but counterpressure is almost always counterproductive. d. Electrodes attached to either side of the spine to provide high-intensity electrical impulses facilitate the release of endorphins.

ANS: D Transcutaneous electrical nerve stimulation does help. Music may be very helpful for reducing tension and certainly can be accommodated by the hospital. Women can stay in a bath as long as they want, although repeated baths with breaks may be more effective than a long soak. Counterpressure can help the woman cope with lower back pain.


Kaugnay na mga set ng pag-aaral

Chapter 7- Life Insurance Underwriting and Policy Issue

View Set

Chapter 07: Legal Dimensions of Nursing Practice

View Set